Sie sind auf Seite 1von 3

MATH 131A, FALL 2006

HOMEWORK 2 SOLUTIONS
1.3, ex. 3: (a) The number of dierent injections from S = {1, 2} into T = {a, b, c} equals the
number of dierent ordered pairs of dierent elements of T, i.e., the number of permutations of
two elements from a set of three elements, which equals 3!/1! = 6.
(b) Suppose f : T S is a surjection. Then f(a) = 1 or f(a) = 2. If f(a) = 1, then f(b) or
f(c) has to equal 2. Therefore, there are three surjections f with f(a) = 1. Similarly, there are
three surjections g : T S such that g(a) = 2. Thus, the number of dierent surjections from
T onto S is six.
1.3, ex. 7: A set T
1
is denumerable if and only if there exists a bijection f : T
1
N. Therefore,
we can take T
2
= N.
1.3, ex. 9: Suppose that S and T are denumerable. By denition, there exists bijections : N S
and : N T. Dene a map f : N S T by
f(n) =

(k), if n = 2k 1 is odd, k 1
(k), if n = 2k is even, k 1.
We claim that f is a surjection. Let x S T be arbitrary. Then x S or x T. Assume
the former. Since is onto, there exists k N such that x = (k). Therefore, by denition of
f, we have x = f(2k 1). If x T the argument is analogous. This shows that f is onto. By
Theorem 1.3.10, it follows that S T is a countable set. However, S T is clearly innite (since
it contains S which is innite), hence it is denumerable, as claimed.
Remark: Observe that f is not an injection if S and T are not disjoint.
1.3, ex. 11: (Induction base) If |S| = 0, then P(S) = {} has exactly 2
0
= 1 element.
(Induction step) Suppose that for every set S with k elements, its power set has 2
k
ele-
ments. Assume that S has k + 1 elements. Let s
0
S be arbitrary. Then S
0
= S \ {s
0
} has
k elements so, by the induction hypothesis, |P(S
0
)| = 2
k
. Let A be an arbitrary subset of S.
There are two possibilities: s
0
A and s
0
A. In the former case, A = A
0
{s
0
}, where
A
0
= A\ {s
0
} P(S
0
). By the induction hypothesis, there are exactly 2
k
possible such sets A
0
.
In the latter case, A P(S
0
), so there are 2
k
such possible As. Overall, there are 2 2
k
= 2
k+1
possibilities for A. In other words, |P(S)| = 2
k+1
. By the PMI, the statement is true for all
n N.
1.3, ex. 12: Suppose that S F(N), i.e., S N is nite. Then there exists n N such that
S N
n
, where N
n
= {1, . . . , n}. Therefore, S P(N
n
), which shows
F(N)

n=1
P(N
n
).
1
2
The opposite inclusion is clearly true, so
F(N) =

n=1
P(N
n
).
Since for each n N, the set P(N
n
) is nite (it has 2
n
elements by the previous exercise), hence
countable, by Theorem 1.3.12 (countable union of countable sets is countable), it follows that
F(N) is countable.
2.1, ex. 7: Suppose the contrary, that is, there exists a rational number r such that r
2
= 3. Since
r Q, we can write r as p/q, for some p Z and q N such that the greatest common divisor
of p and q gcd(p, q) = 1. Since (p/q)
2
= 3, it follows that p
2
= 3q
2
. Therefore, p
2
is divisible
by 3. That can only happen if p itself is divisible by 3. (This can be seen as follows: suppose
p is not divisible by 3. Then p = 3k + s, for some k Z and s {1, 2}. It follows that
p
2
= 9k
2
+ 6ks + s
2
= 3(3k
2
+ 2ks) + s
2
which is not divisible by 3 since s
2
{1, 4}.) Thus
p = 3m, for some m Z. Since p
2
= 3q
2
, we obtain
(3m)
2
= 3q
2
, i.e., 3m
2
= q
2
.
Therefore, 3 divides q
2
, hence it divides q. This means that gcd(p, q) 3, which contradicts our
assumption that gcd(p, q) = 1. Therefore, there exists no r Q such that r
2
= 3.
2.1, ex. 8: (a) Suppose x, y Q. Then x = m/n and y = p/q, for some m, p Z and n, q N.
Therefore,
x + y =
mq + np
nq
Q.
Similarly, xy = mp/nq Q.
(b) Suppose x Q and y R \ Q. If x + y Q, then y = (x + y) + (x) Q, by part (a)
(since x Q). Contradiction! Therefore, x + y R \ Q.
Assume in addition that x = 0. If xy Q, then y = (xy)(1/x) Q, by part (a) and the fact
that 1/x Q. Contradiction! Therefore, xy Q.
2.1, ex. 9: (a) Suppose that x
i
= s
i
+ t
i

2 K are arbitrary. Then


x
1
+ x
2
= (s
1
+ s
2
) + (t
1
+ t
2
)

2 Q,
since s
1
+ s
2
Q and t
1
+ t
2
Q. Furthermore,
x
1
x
2
= (s
1
s
2
+ 2t
1
t
2
) + (s
1
t
2
+ s
2
t
1
)

2 K,
since s
1
s
2
+ 2t
1
t
2
Q and s
1
t
2
+ s
2
t
1
Q.
(b) Suppose x = s + t

2 K and x = 0. Then
1
x
=
1
s + t

2
=
s t

2
s
2
2t
2
=
s
s
2
2t
2
+
t
s
2
2t
2

2 K,
since s/(s
2
2t
2
) Q and t/(s
2
2t
2
) Q (by the previous exercise). Observe that s
2
2t
2
= 0,
since there exists no r Q such that r
2
= 2.
2.1, ex. 18: Assume a b + , for every > 0. Suppose that a b is false, i.e., suppose a > b
and take
0
= (a b)/2. Then
b +
0
= b +
a b
2
=
a + b
2
<
a + a
2
= a
3
contrary to our assumption. Therefore, a b.
2.1, ex. 19: We start with the inequality (a b)
2
0 and obtain:
(a b)
2
0 a
2
2ab + b
2
0
a
2
+ b
2
2ab
2(a
2
+ b
2
) a
2
+ 2ab + b
2
2(a
2
+ b
2
) (a + b)
2

a
2
+ b
2
2

a + b
2

2
,
proving the stated inequality. Observe that the last inequality becomes an equality i the rst
one does, which is i (a b)
2
= 0, i.e., i a = b.
Remark: When a, b 0, this inequality can be written as
a + b
2

a
2
+ b
2
2

1/2
.
The left-hand side is the arithmetic mean of a and b and the right-hand side is their quadratic
mean.

Das könnte Ihnen auch gefallen